K
Khách

Hãy nhập câu hỏi của bạn vào đây, nếu là tài khoản VIP, bạn sẽ được ưu tiên trả lời.

22 tháng 8 2020

thiếu điều kiện là \(x+y+z\le\frac{3}{2}\)bạn nhớ bổ sung 

Sử dụng bất đẳng thức AM-GM cho 3 số ,ta có :

\(\frac{z\left(xy+1\right)^2}{y^2\left(yz+1\right)}+\frac{x\left(yz+1\right)^2}{z^2\left(zx+1\right)}+\frac{y\left(xz+1\right)^2}{x^2\left(xy+1\right)}\ge3\sqrt[3]{\frac{z\left(xy+1\right)^2}{y^2\left(yz+1\right)}.\frac{x\left(yz+1\right)^2}{z^2\left(zx+1\right)}.\frac{y\left(xz+1\right)^2}{x^2\left(xy+1\right)}}\)

\(=3\sqrt[3]{\frac{z\left(xy+1\right)^2.x\left(yz+1\right)^2.y\left(xz+1\right)^2}{y^2\left(yz+1\right).z^2\left(zx+1\right).x^2\left(xy+1\right)}}=3\sqrt[3]{\frac{xyz\left(xy+1\right)^2\left(yz+1\right)^2\left(zx+1\right)^2}{x^2y^2z^2\left(xy+1\right)\left(yz+1\right)\left(zx+1\right)}}\)

\(=3\sqrt[3]{\frac{\left(xy+1\right)\left(yz+1\right)\left(zx+1\right)}{xyz}}=3\sqrt[3]{\left(\frac{xy+1}{x}\right)\left(\frac{yz+1}{y}\right)\left(\frac{zx+1}{z}\right)}\)

Tiếp tục sử dụng bất đẳng thức AM-GM cho 2 số ,ta được :

\(3\sqrt[3]{\left(\frac{xy+1}{x}\right)\left(\frac{yz+1}{y}\right)\left(\frac{zx+1}{z}\right)}=3\sqrt[3]{\left(y+\frac{1}{x}\right)\left(z+\frac{1}{y}\right)\left(x+\frac{1}{z}\right)}\)

\(\ge3\sqrt[3]{\left(2\sqrt{y.\frac{1}{x}}\right)\left(2\sqrt{z.\frac{1}{y}}\right)\left(2\sqrt{x.\frac{1}{z}}\right)}=3\sqrt[3]{\left(2\sqrt{\frac{y}{x}}\right).\left(2\sqrt{\frac{z}{y}}\right).\left(2\sqrt{\frac{x}{z}}\right)}\)

\(=3\sqrt[3]{2.2.2.\sqrt{\frac{y}{x}}.\sqrt{\frac{z}{y}}.\sqrt{\frac{x}{z}}}=3\sqrt[3]{8.\sqrt{\frac{xyz}{xyz}}}=3\sqrt[3]{8}=3.2=6\)

Dấu "=" xảy ra khi và chỉ khi \(x=y=z=\frac{1}{2}\)

Vậy \(P_{min}=6\)đạt được khi \(x=y=z=\frac{1}{2}\)

NV
20 tháng 3 2023

Do \(x^2+y^2+z^2=1\Rightarrow x^2< 1\Rightarrow x< 1\)

\(\Rightarrow x^5< x^2\)

Tương tự ta có: \(y< 1\Rightarrow y^6< y^2\)\(z< 1\Rightarrow z^7< z^2\)

\(\Rightarrow x^5+y^6+z^7< x^2+y^2+z^2\)

\(\Rightarrow x^5+y^6+z^7< 1\)

AH
Akai Haruma
Giáo viên
14 tháng 7 2023

Lời giải:
Áp dụng BĐT Cô-si:

$x^3+1+1\geq 3x$

$y^3+1+1\geq 3y$

$z^3+1+1\geq 3z$

$\Rightarrow x^3+y^3+z^3+6\geq 3(x+y+z)\geq 3.3=9$

$\Rightarrow A=x^3+y^3+z^3\geq 3$ 

Vậy $A_{\min}=3$. Giá trị này đạt tại $x=y=z=1$

NV
7 tháng 5 2023

\(A=\left(x^3+1+1\right)+\left(y^3+1+1\right)+\left(z^3+1+1\right)-6\)

\(A\ge3\sqrt[3]{x^3}+3\sqrt[3]{y^3}+3\sqrt[3]{z^3}-6=3\left(x+y+z\right)-6\ge3.3-6=3\)

\(A_{min}=3\) khi \(x=y=z=1\)

NV
17 tháng 12 2020

Với mọi x;y;z ta luôn có:

\(\left(x+y-1\right)^2+\left(z-\dfrac{1}{2}\right)^2\ge0\)

\(\Leftrightarrow x^2+y^2+2xy-2x-2y+1+z^2-z+\dfrac{1}{4}\ge0\)

\(\Leftrightarrow x^2+y^2+z^2+\dfrac{5}{4}+2xy-2x-2y-z\ge0\)

\(\Leftrightarrow2+2xy-2x-2y\ge z\)

\(\Leftrightarrow2\left(1-x\right)\left(1-y\right)\ge z\)

Dấu "=" xảy ra khi và chỉ khi \(x=y=z=\dfrac{1}{2}\)

 

30 tháng 5 2020

\(x\left(x-z\right)+y\left(y-z\right)=0\)\(\Leftrightarrow\)\(x^2+y^2=z\left(x+y\right)\)

\(\frac{x^3}{z^2+x^2}=x-\frac{z^2x}{z^2+x^2}\ge x-\frac{z^2x}{2zx}=x-\frac{z}{2}\)

\(\frac{y^3}{y^2+z^2}=y-\frac{yz^2}{y^2+z^2}\ge y-\frac{yz^2}{2yz}=y-\frac{z}{2}\)

\(\frac{x^2+y^2+4}{x+y}=\frac{z\left(x+y\right)+4}{x+y}=z-x-y+\frac{4}{x+y}+x+y\ge z-x-y+4\)

Cộng lại ra minP=4, dấu "=" xảy ra khi \(x=y=z=1\)

15 tháng 9 2023

Ta cần chứng minh:\(\dfrac{1}{\sqrt{x+y+xy}}+\dfrac{1}{\sqrt{y+z+yz}}+\dfrac{1}{\sqrt{z+x+zx}}\ge\sqrt{3}\)

Áp dụng bất đẳng thức Bunhiacopxki, ta được:

 

\(\dfrac{1}{\sqrt{x+y+xy}}+\dfrac{1}{\sqrt{y+z+yz}}+\dfrac{1}{\sqrt{z+x+zx}}\ge\dfrac{9}{\sqrt{x+y+xy}+\sqrt{y+z+yz}+\sqrt{z+x+zx}}\)

Mặt khác, ta có:

\(\left(\sqrt{x+y+xy}+\sqrt{y+z+yz}+\sqrt{z+x+zx}\right)^2\le3\left(\left(x+y+xy\right)+\left(y+z+yz\right)+\left(z+x+zx\right)\right)\)

\(\Leftrightarrow\left(\sqrt{x+y+xy}+\sqrt{y+z+yz}+\sqrt{z+x+zx}\right)^2\le3\left(6+xy+yz+zx\right)\)Lại có:

\(xy+yz+zx\le\dfrac{\left(x+y+z\right)^2}{3}=\dfrac{9}{3}=3\)

\(\Rightarrow\left(\sqrt{x+y+xy}+\sqrt{y+z+yz}+\sqrt{z+x+zx}\right)^2\le3\left(6+3\right)=27\)

15 tháng 9 2023

\(\Rightarrow\sqrt{x+y+xy}+\sqrt{y+z+yz}+\sqrt{z+x+zx}\le3\sqrt{3}\)

\(\Rightarrow\dfrac{9}{\sqrt{x+y+xy}+\sqrt{y+z+yz}+\sqrt{z+x+zx}}\ge\dfrac{9}{3\sqrt{3}}=\sqrt{3}\)

Do đó \(\dfrac{1}{\sqrt{x+y+xy}}+\dfrac{1}{\sqrt{y+z+yz}}+\dfrac{1}{\sqrt{z+x+zx}}\ge\sqrt{3}\)

Dấu bằng xảy ra \(\Leftrightarrow x=y=z=1\).

14 tháng 5 2021

hiiiii

29 tháng 7 2021

rg